Intégrale AMM12338

Fin de partie
Modifié (June 2022) dans Analyse
Problème AMM12338
Montrer que $\displaystyle \int_0^\infty \frac{\cos x-1}{x\left(\text{e}^x-1\right)}dx=\frac{1}{2}\ln\left(\frac{\pi}{\sinh(\pi)}\right)$
PS.
Je pense avoir fini de calculer cette intégrale. Je ne suis pas trop content de la méthode mais elle fonctionne.

PS2:
Ne cherchez pas ce problème sur l'excellent site de Tauraso il ne l'a pas encore traité à la date du 7 juin 2022 (16h00).

Réponses

  • Bon ça commence mal, j’ai tenté de “simplifier” en remplaçant le $\exp(x) - 1$ par $\exp (x) $ pour voir si ça allait me donner une bonne inspiration et je suis tombé sur la série harmonique. 
    ---> I believe in Chuu-supremacyhttps://www.youtube.com/watch?v=BVVfMFS3mgc <---
  • bd2017
    Modifié (June 2022)
    Tous les calculs qui suivent se justifient:
    $f(a)=\int_0^\infty \dfrac{\cos (a x)-1}{(e^x-1) x}dx$
    $f'(a)=\int_0^\infty \dfrac{\sin (a x)}{1-e^x }dx $
    $f'(a)=\sum_{n\geq 1} \int_0^\infty \sin (a x)e^{-n x}dx=-\sum_{n\geq 1}\dfrac{a}{a^2+n^2}=\dfrac{1}{2 a} - \dfrac{\pi  \coth (\pi  a)}{2 }$
    $f(a)=\dfrac{\log (a )}{2}-\dfrac{1}{2} \log (\sinh (a \pi ))+cste$

    On trouve la constante avec $a=0$ puis la valeur de $f(1).$
    Edit  j'ai oublié de recopier le $-1,$  je l'ai donc ajouté.
     
  • Mais c'est $\cos(x) - 1$ au numérateur. L'auteur se serait-il fourvoyé ? 
    ---> I believe in Chuu-supremacyhttps://www.youtube.com/watch?v=BVVfMFS3mgc <---
  • Non, mais rajouter un -1 ne change rien au raisonnement de bd2017.
  • Fin de partie
    Modifié (June 2022)
    Il n'y a aucune erreur, le résultat annoncé est correct.
    @Bd2017: Jusqu'à ta deuxième ligne mon calcul est le même que le tien. (à un changement de variable près).
  • @Positif: Si tu enlèves le $-1$ au numérateur l'intégrale est divergente.
  • @FDp Et  ensuite je passe aux séries. Je me doute qu'à partir de là tu as fait des manipulations sur les intégrales ? 
     
  • Fin de partie
    Modifié (June 2022)
    \begin{align}J&=\int_0^\infty \frac{\cos x-1}{x\left(\text{e}^x-1\right)}dx\\
    &\overset{u=\text{e}^{-x}}=-\int_0^1 \frac{\cos(\ln u)-1}{\ln u\left(\frac{1}{u}-1\right)}\times \frac{1}{u}du\\
    &=-\int_0^1 \frac{\cos(\ln u)-1}{\ln u\left(1-u\right)}du\\
    &=\int_0^1\int_0^1\frac{\sin(a\ln u)}{1-u}dadu\\
    &=\int_0^1 \bigg(\sum_{n=0}^\infty \frac{(-1)^na^{2n+1}}{(2n+1)!}\underbrace{\int_0^1 \frac{\ln^{2n+1} u}{1-u}du}_{-(2n+1)!\zeta(2n+2)}\bigg)da\\
    &=-\int_0^1\left(\sum_{n=0}^\infty(-1)^n\zeta(2n+2) a^{2n+1}\right)da\\
    &=-\sum_{n=0}^\infty \frac{(-1)^{n-1}\zeta(2n+2)}{2n+2}\\
    &\overset{k=n+1}=\frac{1}{2}\sum_{n=1}^\infty \frac{(-1)^k\zeta(2k)}{k}\\
    \end{align}
    Pour terminer il faut calculer cette série et cela se fait à partir du développement en produit infini de $\sinh(z)$
    PS.
    C'est laid de mon point de vue, mais j'ai des doutes que je puisse trouver une solution complète sans le recours à une série.
  • jean lismonde
    Modifié (June 2022)
    Bonjour
    On part du produit infini eulérien : $\displaystyle \frac{\sinh(\pi x)}{\pi x} = (1+x^2)(1+\frac{x^2}{2^2})\cdots(1+\frac{x^2}{p^2})\ldots$
    on considère les logarithmes népériens de chaque facteur (tous positifs) que nous dérivons par rapport à x
    il vient la série fractionnaire quelle que soit x  :
    $\displaystyle \frac{\pi}{2x\tanh(\pi.x)} - \frac{1}{2x^2} = \frac{1}{1^2+x^2} + \frac{1}{2^2+x^2}+\cdots+ \frac{1}{p^2+x^2}+\ldots$
    que nous pouvons développer en série polynomiale (après avoir multiplié par x)
    $\displaystyle \frac{\pi}{2\tanh(\pi x)} - \frac{1}{2x} = x.Z_2 -  x^3Z_4 + x^5Z_6 - \ldots$
    avec $Z_{2p}$ la fonction Zéta de Riemann pour les entiers pairs de la variable
    nous considérons maintenant la relation intégrale liée à la fonction eulérienne Gamma (avec p > 0) :
    $\displaystyle \int_0^{+\infty}e^{-pt}.t^xdt=\frac{\Gamma(x+1)}{p^{x+1}}$
    et pour $ x = n$ entier naturel et en considérant le développement en série de $\displaystyle \frac{e^{-t}}{1-e^{-t}}$ il vient :
    $\int_0^{+\infty}\frac{t^n}{n!}\frac{dt}{e^t-1} = Z_{n+1}$
    et si nous considérons le développement polynomial de $\sin(tx)$ il vient :
    $\displaystyle \int_0^{+\infty}\frac{\sin(tx)}{e^t - 1}dt= xZ_2 - x^3Z_4 + x^5Z_6 - \ldots= \frac{\pi}{2\tanh(\pi.x)} - \frac{1}{2x}$
    nous pouvons primitiver sous le signe intégral avec une constante d'intégration égale à 1 pour assurer la continuité de la fraction en zéro
    il vient en conséquence : $$\int_0^{+\infty}\frac{1-\cos(tx)}{t(e^t - 1)}dt = \frac{1}{2}\ln\frac{\sinh(\pi.x)}{\pi.x}$$Cordialement.
  • Area 51
    Modifié (June 2022)
    Résultat marrant, si bien qu'il est tentant d'invoquer le produit eulérien du sinus hyperbolique :
    $$\frac{\pi}{\sinh \pi z} = \frac{1}{z} \prod_{k=1}^{+\infty} \frac{k^2}{z^2+k^2}$$
    En se souvenant que la transformée de Fourier d'une porte est un sinus cardinal, on obtient :
    $$\int_0^{+\infty} \frac{\cos x - 1}{e^x - 1} \frac{dx}{x} = -\int_0^{+\infty} \frac{\sin \frac{x}{2}}{e^x - 1} \bigg( \int_{-\frac{1}{2}}^{\frac{1}{2}} e^{ixy} \, dy \bigg) dx = -\frac{1}{2i} \int_0^{+\infty} \int_{-\frac{1}{2}}^{\frac{1}{2}} \sum_{k=1}^{+\infty} \Big[ e^{- \big( k-iy-\frac{i}{2} \big) x} - e^{- \big( k-iy+\frac{i}{2} \big) x} \Big] dy \, dx$$
    où ne demeurent que d'honnêtes exponentielles. Maintenant, Fubini plie la game :
    $$\begin{eqnarray} \int_0^{+\infty} \frac{\cos x - 1}{e^x - 1} \frac{dx}{x} & = & -\frac{1}{2i} \sum_{k=1}^{+\infty} \int_{-\frac{1}{2}}^{\frac{1}{2}} \Bigg( \frac{1}{k-iy-\frac{i}{2}} - \frac{1}{k-iy+\frac{i}{2}} \Bigg) dy \\ & = & -\frac{1}{2} \sum_{k=1}^{+\infty} \bigg[ \log \Big( k-iy-\frac{i}{2} \Big) - \log \Big( k-iy+\frac{i}{2} \Big) \bigg]_{-\frac{1}{2}}^{\frac{1}{2}} \\ & = & -\frac{1}{2} \sum_{k=1}^{+\infty} \big[ \log (k-i) - 2 \log k + \log (k+i) \big] = \frac{1}{2} \log \prod_{k=1}^{+\infty} \frac{k^2}{1+k^2} \end{eqnarray}$$
  • L2M
    L2M
    Modifié (June 2022)
    Et si on essaye d'écrire cette intégrale en fonction de zêta de Hurwitz $\zeta(z,q)$ (ma fonction préférée).
    Soit la fonction $\displaystyle g(a,b) = \int_0^\infty \frac{\cos(a x)-1}{x^b(e^x-1)}dx$. $a, b\in \mathbb{R}$.
    On la dérive par rapport à $a$. $\displaystyle \partial_a g(a,b) = - \int_0^\infty \frac{x^{1-b}\sin(a x)}{e^x-1}dx = -\Im\left( \int_0^\infty \frac{x^{1-b}e^{-(1-ia)x}}{1-e^{-x}}dx\right) = -\Im \zeta(2-b,1-ia)$.
    On sait d'après la dérivée de zêta de Hurwitz que, $\displaystyle \zeta(2-b,1-ia) = -\frac{i}{1-b}\partial_a \zeta(1-b,1-ia).$
    Ainsi, $\displaystyle g(a,b) = \Im \left( \frac{i}{1-b} \zeta(1-b,1-ia)\right)+ cst.$
    Puisque, $\displaystyle g(0,b)=0=\Im \left( \frac{i}{1-b} \zeta(1-b,1)\right) + cst$, alors $\displaystyle cst=-\frac{\zeta(1-b)}{1-b} $, $\zeta(z,1)=\zeta(z)$.
    Ce qui donne, $$g(a,b) = \Im\frac{i\zeta(1-b,1-ia)}{1-b} - \frac{\zeta(1-b)}{1-b}$$
    Maintenant, on tend $a\to 1$ et $b\to 1$. (Il faut ajouter que $\zeta(0,1-ia)=ia-1/2$ et $\zeta(0)=-1/2$)
    $$g(1,1) = \int_0^\infty \frac{\cos x-1}{x\left(\text{e}^x-1\right)}dx= \Im \left(i\zeta'(0,1-i)\right) -\zeta'(0).$$
    c-à-d, $$ \int_0^\infty \frac{\cos x-1}{x\left(\text{e}^x-1\right)}dx = \Re \left(\zeta'(0,1-i)\right) -\zeta'(0).$$
    On sait déjà que : $\displaystyle \zeta'(0)=-\frac{1}{2} log(2 π)$, donc $$\Re \left(\zeta'(0,1-i)\right) =\frac{1}{2}\ln\left(\frac{\pi}{\sinh(\pi)}\right) - \frac{1}{2} log(2 π).$$ $$\zeta'(0,1-i) = - \frac{1}{2}\ln\left(2\sinh\pi\right)+i\ ???$$
    Vérification.
    $\displaystyle \zeta'(0,1-i)=-1.56986... + i 0.30164...$  voir ici.
    $\displaystyle \frac{1}{2}\ln\left(2\sinh\pi\right)=1.5698617325065290806655465679...$  voir ici.

    Remarque importante. La partie imaginaire de $\zeta'(0,1-i)$ est liée aux valeurs de zêta aux entiers impaires, alors il est presque impossible de la calculer de façon close comme on l'a fait pour sa partie réelle.
Connectez-vous ou Inscrivez-vous pour répondre.